6
$\begingroup$

Let $S = (\Omega,\leq)$ be an uncountable dense total order, such that for all positive integers $m$ and all finite ordered sequences $a_1 < a_2 < \ldots < a_m$ and $b_1 < b_2 < \ldots < b_m$, we have an order automorphism of $S$ mapping the former sequence to the latter.

Does there exist a proper subset $\Omega' \subset \Omega$ such that the induced total order $S'$ is order isomorphic to $S$ ?

If so, can $\Omega'$ be chosen to be an interval ?

(I guess the answer will follow from some general theorem on total orders, and I would love to know what that is.)

Thanks !

$\endgroup$
2
  • $\begingroup$ When you say interval, do you mean a bounded interval, or also a ray? $\endgroup$
    – Asaf Karagila
    Jan 23, 2020 at 13:30
  • $\begingroup$ @AsafKaragila : for me, rays are also intervals. $\endgroup$
    – THC
    Jan 23, 2020 at 13:36

1 Answer 1

8
$\begingroup$

For the first question, the answer is YES, even if we just assume that $S$ is not rigid. Indeed, fix $f\in\mathrm{Aut}(S)$ and $a\in S$ such that $a<f(a)$. Then $\bigl(\mathrm{id}\restriction(-\infty,a]\bigr)\cup\bigl(f\restriction(a,+\infty)\bigr)$ is an isomorphism of $S$ to its proper suborder $(-\infty,a]\cup(f(a),+\infty)$.

For the second question, the answer is NO in general. One counterexample is the lexicographic product $S=(\omega_1^*+\omega_1)\times\mathbb Q$.

To see that $S$ has the required property (which, by the way, is what model theorists call being strongly $\omega$-homogeneous), notice that any given $a_1,\dots,a_m,b_1,\dots,b_m$ are included in a countable open subinterval of $S$. Such a subinterval must be isomorphic to $\mathbb Q$, which is strongly $\omega$-homogeneous, and an automorphism of the subinterval extends to an automorphism of $S$ by extending it with the identity.

On the other hand, if $S'\subseteq S$ is isomorphic to $S$, it has upwards and downwards cofinality $\omega_1$, which can only happen if it is an (upwards and downwards) cofinal subset of $S$. Thus, it cannot be a proper subinterval.

$\endgroup$

Your Answer

By clicking “Post Your Answer”, you agree to our terms of service and acknowledge you have read our privacy policy.

Not the answer you're looking for? Browse other questions tagged or ask your own question.